Math, asked by Jegdheesh, 1 year ago

plssssssssszzzzz answer my question with explanation

Attachments:

Answers

Answered by Anonymous
1
I think option C because in this both no. are irrational

shubhamtiwari31: u have to find an irrational no. they are not the options
Anonymous: ooo I am very sorry
shubhamtiwari31: no prblm..
Similar questions